Exam 3 Gero

Ace your homework & exams now with Quizwiz!

Which condition is included in the diagnosis of chronic obstructive pulmonary disease (COPD)? a. Bronchial asthma b. Histoplasmosis c. Bacterial pneumonia d. Mycobacterium tuberculosis

a. Bronchial asthma COPD includes asthma, chronic bronchitis, and emphysema. Neither pneumonia histoplasmosis nor tuberculosis is considered among those associated with COPD.

An older has adult diagnosed with COPD wants to perform self-care activities. Which instruction should the nurse include in client teaching to help achieve this goal? a. Bathe and eat slowly with periodic rest. b. Walk short distances without oxygen. c. Perform all activities of daily living (ADLs) and then rest. d. Bathe right after eating, and then rest.

a. Bathe and eat slowly with periodic rest. A person with COPD can perform self-care tasks if allowed plenty of time to accomplish them and to take breaks for rest. The client can potentially benefit more from longer periods of exercise supplemented with oxygen than from short excursions without oxygen. A plan to rest in the future after the self-care task of performing ADLs or bathing after eating is accomplished does not compensate for the deprivation of rest when she needs it.

Which statements made by a couple who have recently retired support the nurse's evaluation that the transition to retirement has been a successful one? (Select all that apply.) a. "I'm afraid we can't make it that weekend; we'll be visiting the grandchildren." b. "I'm not accustomed to sharing my kitchen with anyone else." c. "I cook; he does the dishes." d. "I often wonder if my buddies from the plant miss me." e. "We have found a few painless ways to reduce our monthly expenses."

a. "I'm afraid we can't make it that weekend; we'll be visiting the grandchildren." c. "I cook; he does the dishes." e. "We have found a few painless ways to reduce our monthly expenses." The correct options demonstrate a sense of purpose, cooperation, and adjustment to the realities of retirement and so that the transition has been a successful one. The remaining options suggest that the couple is not adjusting well to having each other around so much and involved in activities they previously used as being theirs alone. Feeling lonely or abandoned is also a sign of ineffective adjustment. A transition into a retirement rather than an abrupt one is also a barrier to a successful adjustment into retirement.

Which of the following statements is true about dysarthria? a. A nasogastric tube may be necessary. b. Stems from severe rheumatoid arthritis. c. Physical therapy can be beneficial. d. Can affect the balance.

a. A nasogastric tube may be necessary. Dysarthria is a speech disorder caused by a weakness or incoordination of the speech muscles. It occurs as a result of central or peripheral neuromuscular disorders that interfere with eating and a nasogastric (NG) tube may be considered to provide artificial nutrition; it does not affect intelligence. It does not stem from rheumatoid arthritis. Occupational therapy can help. Dysarthria does not affect balance.

Which situation would the nurse identify as placing a client at high risk for caregiver abuse? a. An adult child quits her job to move in and care for a parent with severe dementia. b. An elderly man with severe heart disease resides in a personal care home and is visited frequently by his adult child. c. An elderly parent with limited mobility lives alone and receives help from several adult children. d. An elderly woman who cares for her husband who is in early stages of Alzheimer's disease and has a network of available support persons.

a. An adult child quits her job to move in and care for a parent with severe dementia. In this situation, the adult child has given up her usual role as well as moved her place of residence to care for her parent. Caring for someone with severe dementia is very stressful, requiring almost 24-hour vigilance to ensure safety and meet needs. This situation places the caregiver at high risk for stress and abuse.

When planning care for a client that has a history of alcohol abuse, the nurse recognizes which of the following medication(s) will interact with alcohol? (Select all that apply.) a. Analgesics b. Antibiotics c. Antidepressants d. Antipyretics e. Antipsychotics

a. Analgesics b. Antibiotics c. Antidepressants Many drugs that older adults use for chronic illnesses cause adverse effects when combined with alcohol. Medications that interact with alcohol include analgesics, antibiotics, antidepressants, antipsychotics, benzodiazepines, H2-receptor antagonists, nonsteroidal anti-inflammatory drugs (NSAIDs), and herbal medications (e.g., Echinacea, valerian). Acetaminophen taken on a regular basis, when combined with alcohol, may lead to liver failure. Alcohol diminishes the effects of oral hypoglycemics, anticoagulants, and anticonvulsants. All older adults should be given precise instructions regarding the interaction of alcohol with their medications.

While awaiting the imminent death of her sister, an older woman makes arrangements to bury her sister in the survivor's home state because she cannot reach the other family members. Which step should the nurse implement? a. Ask questions, including questions about the location of her sister's family. b. Instruct this woman that this is not her decision to make. c. Try to contact the family to inform them of the decision. d. Question her about holding behaviors that she will want.

a. Ask questions, including questions about the location of her sister's family. In a countercoping intervention, the nurse assists the older adult to cope with the loss by collecting information and encouraging her to avoid acting on impulse. The older adult can arrive at a hasty decision when not effectively coping with grief; therefore the nurse acts to help restore some control for the bereaved and helps avoid a decision that might be regretted later. The nurse can ask the older adult if she believes that this is her decision to make but avoids informing her that it is not. The nurse avoids acting without the sister's knowledge; the nurse does not have the right to impose personal feelings on the family or on the client. At a time when the older adult is acting impulsively, it can be reasonable to have her cool off; however, the woman has made a hasty decision and needs help to resolve that issue. In addition, the nurse can be considered insensitive for asking about this issue before the client is dead.

Which information will the nurse include when discussing tips for reducing caregiver stress with family caregivers? (Select all that apply.) a. Attend to your own needs. b. Set long-term goals. c. Limit tapping your social resources for assistance. d. Use community resources. e. Take time to relax and exercise. f. Educate yourself about the disease or medical condition.

a. Attend to your own needs. d. Use community resources. e. Take time to relax and exercise. f. Educate yourself about the disease or medical condition. Caregiver burden is defined as the negative psychological, economic, and physical effects of caring for a person who is impaired. Whereas not all caregivers experience stress and caregiver burden, the circumstances that are more likely to cause problems with caregiving include competing role responsibilities, advanced age of the caregiver, high-intensity caregiving needs, insufficient resources, financial difficulty, poor self-reported health, living in the same household with care recipient, dementia of the care recipient, length of time caregiving, and prior relational conflicts between the caregiver and care recipient. Taking care of oneself, asking for assistance, and education are ways to reduce caregiver stress. Positive benefits of caregiving may include enhanced self-esteem and well-being, personal growth and satisfaction, and finding or making meaning through caregiving (Sorrell, 2014). The caregiver should set realistic goals, not long-term goals. It is encouraged for caregivers to tap their social resources for assistance.

A nurse is teaching an older adult who is experiencing an acute attack of gout. Which of the following should the nurse include in the teaching? a. Avoid foods high in purine. b. Encourage the client to take in 1 L of fluid daily. c. Consume one glass of red wine daily. d. Recommend that the client eat 12 to 16 ounces of foods high in protein such as red meat.

a. Avoid foods high in purine. A person who is having an acute attack of gout should avoid foods that are high in purine, take in 2 L of fluid daily, avoid alcohol and only have 4 to 6 ounces of foods high in protein daily.

Which nursing assessment findings are specific signs of sexual abuse of an older adult? (Select all that apply.) a. Bruises or scratches in the genital area b. Torn undergarments or presence of blood c. Unexplained lacerations in various stages of healing d. Fractures inconsistent with functional ability e. Bruises or scratches in the breast area

a. Bruises or scratches in the genital area b. Torn undergarments or presence of blood e. Bruises or scratches in the breast area Bruises or scratches in the genital area, torn undergarments or presence of blood, and bruises or scratches in the breast area are all indications that a female elder has been the victim of sexual abuse. The remaining options are signs of physical abuse of an elder, not sexual abuse.

Which intervention to manage wandering in clients in a long-term care facility should be implemented? (Select all that apply.) a. Camouflaging doorways b. Close observation to identify the person's individual patterns c. Engaging the person in social interactions d. Using physical restraints to prevent wandering to maintain safety e. Providing enclosed pathways for walking

a. Camouflaging doorways b. Close observation to identify the person's individual patterns c. Engaging the person in social interactions e. Providing enclosed pathways for walking Restraints are not an effective intervention for wandering. Although they might physically prevent the person from wandering, restraints have many potential negative consequences and client harm associated with their use. Environmental modifications such as camouflaging doorways and providing enclosed pathways, close observation to identify the person's individual patterns, and engaging the person in social interactions are all interventions that are effective strategies to manage wandering.

An older has adult diagnosed with heart failure (HF) reports increasing dyspnea over 2 days. Which of the following should the nurse assess to help determine whether the client has adhered to prescribed therapy? (Select all that apply.) a. Check for peripheral edema. b. Ask about bowel pattern. c. Auscultate the lungs bilaterally. d. Compare current weight to baseline. e. Determine coughing frequency. f. Assess diet over last 48 hours.

a. Check for peripheral edema. c. Auscultate the lungs bilaterally. d. Compare current weight to baseline. f. Assess diet over last 48 hours. Standard HF therapy includes taking medications as prescribed with a low-sodium diet to control total body fluids. Usually, dyspnea in a client with HF is due to hypervolemia, which occurs after a lapse in adherence to the standard HF therapy. The nurse checks the client's extremities for edema because peripheral edema is a clinical indicator of hypervolemia. If the client is nonadherent with therapy, then the nurse is more likely to find peripheral edema than with an adherent client. Hypervolemia can also be due to worsening HF. The nurse listens to the client's lungs to assess for pulmonary edema as a cause for the client's dyspnea. Pulmonary edema can be caused by hypervolemia from nonadherence to therapy or from worsening HF. The nurse compares the client's weight to his baseline to determine whether the client has experienced a sudden weight gain, which would be indicative of hypervolemia. Assessing the client's diet over the last 48 hours can provide clues about a potential cause of the client's dyspnea. If the client increases the dietary sodium by eating pizza, pickles, and processed food, among others, the client is likely to experience a sudden increase in total body fluid, which can cause the client's dyspnea. Although older adults with HF complain of anorexia, bowel habits are not as likely to be affected by hypervolemia as is the appetite. Coughing is a nonspecific, nonsensitive indicator of pulmonary edema.

The nurse is educating an older adult on foods high in calcium. Which foods should the nurse include? (Select all that apply.) a. Chinese cabbage b. Soy milk c. Cheese pizza d. Whole wheat bread e. Almonds

a. Chinese cabbage b. Soy milk c. Cheese pizza e. Almonds Chinese cabbage, soy milk, almonds, and cheese pizza have all been identified as foods that are high in calcium. Whole wheat bread contains calcium; however, it is not a high calcium-rich food.

The nurse is evaluating the plan of care for an older adult who is alcohol-dependent. Which client documentation indicates the need for follow-up nursing interventions by the nurse? a. Client states that he intends to decrease his alcohol consumption. b. Client arrives at his group session on time and well-groomed. c. Client states, "I am an alcoholic because I drink 10 beers a day." d. Client states that he understands that he needs continued treatment.

a. Client states that he intends to decrease his alcohol consumption. When the client states that he or she intends to decrease alcohol consumption, this response indicates that the client continues to believe that his or her alcohol consumption is under his or her control. If the client arrives at a group session on time and is well-groomed, taking pride in his or her appearance and participating in a group activity are positive signs. Acknowledging that he or she has a problem is a positive sign; older adults cannot be helped until the problem is acknowledged. Acknowledging the need for continuing treatment is a positive sign.

Which of the following is the most important goal in the nursing plan of care to decrease the frequency of hospitalizations for acute exacerbations of heart failure (HF) in older adults? a. Control fluid balance. b. Control blood pressure. c. Prevent deconditioning. d. Maintain client safety.

a. Control fluid balance. The most important goal for keeping a client who has HF out of the hospital is to control total body fluid; hypervolemia aggravates HF by increasing the blood volume and making the heart work harder. Controlling total body fluid also helps prevent dyspnea and hypertension, maintain physical activity, improve rest and sleep, and promote nutrition for optimal health and wellness. Controlling the blood pressure is an important part of HF therapy; however, fluid volume status is implicated more often in those hospitalized with HF. Preventing deconditioning is an important yet challenging goal for clients with HF, but it is not frequently implicated in those hospitalized with HF. Maintaining client safety is an important goal for any client, but it is not commonly implicated as a cause of hospitalization for those with HF.

A nurse is educating a group of older adults on the impact of lifestyle changes on hypertension. The nurse includes which of the following in the education? (Select all that apply.) a. Discussion of smoking cessation techniques b. Strategies for reaching a healthy weight c. Techniques to incorporate more physical activity into the daily routine d. The actions of calcium channel blocker medications on hypertension e. The importance of adhering to pharmacological regimens for treatment of hypertension

a. Discussion of smoking cessation techniques b. Strategies for reaching a healthy weight c. Techniques to incorporate more physical activity into the daily routine Nursing actions include promoting an overall healthy lifestyle: cease or avoid smoke and other pollutants, a balanced diet (e.g., DASH diet), a healthy weight, regular exercise, and emotional modulation. The other options are related to pharmacological treatment of hypertension.

Which one of the following older adults has the highest economic risk in retirement at the beginning of retirement? a. Divorced woman who has lived in this country for 3 years b. Male veteran who is an above-the-knee amputee and was a teacher c. Female widow who is a primary care nurse practitioner d. Male who immigrated from China and designs computer software

a. Divorced woman who has lived in this country for 3 years The older divorced woman who has lived in this country for 3 years has three factors associated with economic risk in retirement: (1) female sex, (2) divorced, and (3) immigrant with inadequate time to be eligible for government-sponsored retirement benefits. The older male veteran, an above-the-knee amputee, was a teacher and has one risk factor—disability. The older female widow and primary care nurse practitioner has two risk factors: (1) female gender and (2) widowhood. The older man who emigrated from China and designs computer software has one risk factor—immigrant.

Those who cope less effectively may exhibit which of the following? (Select all that apply.) a. Engages in avoidance b. Confronts reality c. Is demanding d. Is rigid e. Is pessimistic

a. Engages in avoidance c. Is demanding d. Is rigid e. Is pessimistic Those who cope less effectively have few if any of these abilities listed by Weisman. They tend to be more rigid and pessimistic, are demanding, and are given to emotional extremes and avoidance. Those who cope well confront reality.

Which disease has become known as the "great imitator?" a. Human immunodeficiency virus (HIV) b. Acquired immunodeficiency syndrome (AIDS) c. Alzheimer disease d. Schizophrenia

a. Human immunodeficiency virus (HIV) The compromised immune system of an older individual makes him or her more susceptible to HIV or AIDS than a younger person. AIDS in older adults has been called the "great imitator"; many of the symptoms, such as fatigue, weakness, weight loss, and anorexia, are common to other disease conditions and may be attributed to normal aging. Alzheimer disease and schizophrenia are not known as the "great imitator."

Which is a healthy practice recommended for a person at risk for Osteoarthritis (OA)? a. Milk and orange juice at breakfast; cheese pizza at lunch; spaghetti served with spinach covered with melted cheese for dinner; and ice cream for dessert b. Long-term estrogen administration as adjunct therapy c. A bisphosphonate medication taken with a snack just before bedtime d. Coffee, raisin bran and milk, and sausage at breakfast; a can of cola and a hot dog on a high-fiber bun at lunch; cocktails before dinner; steak with brown rice, celery, and red wine for dinner

a. Milk and orange juice at breakfast; cheese pizza at lunch; spaghetti served with spinach covered with melted cheese for dinner; and ice cream for dessert These foods in these quantities supply 1204 mg of calcium. Administering estrogen can increase the risk of cancer and heart disease. Because of the seriousness of the risk for esophageal erosion, bisphosphonates must be taken with a full glass of water on an empty stomach after awakening. Afterward, the client must sit upright and refrain from eating or drinking for 30 minutes Alcohol and high amounts of protein and salt inhibit calcium uptake, whereas caffeine, excess fiber, and phosphorus (in the cola) promote calcium excretion.

Which assessment is typical for a client with osteoarthritis (OA)? a. Narrow joint spaces with crepitus b. Effects in symmetrical joints c. Morning stiffness for at least an hour d. Swelling from excess synovial fluid

a. Narrow joint spaces with crepitus The joint of an older adult with OA is narrower than a normal joint, and as the disease advances, crepitus is palpable over the joint. The joint narrows as it degenerates, and crepitus occurs as the articulating surfaces of the bone abnormally move against each other. Disease in symmetrical joints is characteristic of RA. Morning stiffness lasting for 1 hour or more is characteristic of polymyalgia rheumatica. Swelling from excessive synovial fluid is characteristic of RA.

The nurse understands that heart disease's modifiable risk factors include which of the following? (Select all that apply.) a. Obesity b. Hypertension c. Uncontrolled diabetes d. Macular degeneration e. Nicotine use f. Allergies

a. Obesity b. Hypertension c. Uncontrolled diabetes e. Nicotine use Age, hypertension, cigarette smoking, obesity, inactivity, dyslipidemia, and diabetes are all risk factors for the development of heart disease. Neither macular degeneration nor allergies are generally associated with heart disease.

Alcohol diminishes the effects of what type(s) of medications? (Select all that apply.) a. Oral hypoglycemic b. Anticoagulant c. Anticonvulsants d. Tricyclic antidepressants e. Acetaminophen

a. Oral hypoglycemic b. Anticoagulant c. Anticonvulsants Alcohol diminishes the effects of oral hypoglycemics, anticoagulants, and anticonvulsants. Alcohol increases the effect of tricyclic antidepressants. Acetaminophen taken on a regular basis, when combined with alcohol, may lead to liver failure.

Which assessment parameter should the nurse use to differentiate between delirium and depression in an older adult? a. Orientation b. Activity c. Course over the morning hours d. Psychomotor activity

a. Orientation Qualities about the client's orientation are a good method for the nurse to use for distinguishing between delirium and depression; in delirium, orientation is usually impaired, and in depression, orientation is normal. Activity can vary throughout the day and is not a good indicator. Delirium tends to be worse at night, and depression tends to be worse in the morning. The nurse avoids using qualities about the client's psychomotor activities to distinguish between delirium and depression in an older adult; psychomotor activities in both disorders are highly variable and make distinctions difficult.

The nurse working in a long-term care facility completes morning assessment on a new postoperative client and notes a change in cognitive status from the previous day? The nurse recognizes which of the following as a precipitating factor for delirium? (Select all that apply.) a. Poor nutrition b. Poor sleep habits c. Dehydration d. Female gender e. History of dementia

a. Poor nutrition b. Poor sleep habits c. Dehydration e. History of dementia Precipitating factors for the development of delirium include existing cognitive impairment, sleep deprivation or disturbance, dehydration, and poor nutrition. Gender is not a significant factor.

Which of the following characteristics are associated with acute grief? (Select all that apply.) a. Preoccupation with the loss of a loved one b. Waves of grief or distressing emotion c. Prolonged inability to sleep after a loss d. Exacerbations of grief on specific dates e. Change in attitude toward the future loss f. Inability to perform simple self-care tasks

a. Preoccupation with the loss of a loved one b. Waves of grief or distressing emotion f. Inability to perform simple self-care tasks In acute grief, the bereaved is preoccupied with the deceased or the loss in a manner similar to daydreaming, combined with a sense of unreality. Specific activities, items, people, or other things can trigger an overwhelming pain in acute grief. The bereaved can be incapacitated by acute grief, making simple tasks such as dressing nearly impossible to complete or taking much longer to complete a task. Chronic grief is characterized by prolonged insomnia and an extended period of inhibited activities and suboptimal performance. Chronic grief is characterized by periods of pain exacerbated on specific dates such as anniversaries, birthdays, and holidays, among others. Anticipatory grief is characterized by a change in attitude toward the individual who is about to die when the death does not occur as planned.

An older adult has a wound infection 5 days after a below-the-knee amputation brought about by diabetes mellitus. Which of the following is the nurse's priority intervention to prevent cognitive dysfunction and postoperative complications in this older adult? a. Remove invasive devices as soon as possible. b. Minimize the administration of opioid analgesics. c. Allow for self-care and independent activities. d. Administer short-acting benzodiazepines as needed.

a. Remove invasive devices as soon as possible. To help prevent cognitive dysfunction, postoperative complications, and an increased risk of morbidity and mortality, the nurse recognizes that the risk factors this older adult has for delirium include stressors, infection, and surgery; therefore, to prevent cognitive decline and additional postoperative complications, the nurse promptly removes invasive devices such as intravenous infusions, urinary catheters, and wound drains. Removing these devices not only reduces the risk of infection, thromboembolic events, blood loss, injury, and fluid imbalance, but it also serves to promote mobility, promote a sense of control for the client, and reduce the types of situations that can frighten the client or that the client can misinterpret. Poor pain management can contribute to delirium in older clients. A client with multiple stressors and risk factors for delirium needs additional nursing care and attention to provide a calming, caring therapeutic environment. The nurse must assess the client's functional status before allowing self-care and independent activities. In addition, this older adult is likely to need extensive physical therapy to maintain mobility. Benzodiazepines are a poor pharmacological choice for older adults for sedation or sleep; they can contribute to delirium, are highly addictive, and can cause rebound insomnia if suddenly withdrawn.

Which of the following are common side effects of Parkinson's disease (PD) and the medications used to treat it? (Select all that apply.) a. Sleep disorders b. Dyskinesias c. Dystonia d. Nausea e. Depression

a. Sleep disorders b. Dyskinesias c. Dystonia e. Depression Medication therapy is complicated and must be closely supervised. Hypotension, dyskinesias (involuntary movements), dystonia (lack of control of movement), hallucinations, sleep disorders, and depression are common side effects of both the disease and the medications used to treat it. Nausea is not a side effect of PD.

Which is(are) potential result(s) of end-organ damage from chronic hypertension? (Select all that apply.) a. Stroke b. Cardiac disease c. Renal insufficiency d. Atrial fibrillation e. Isolated systolic hypertension f. Familial hypercholesterolemia

a. Stroke b. Cardiac disease c. Renal insufficiency d. Atrial fibrillation Older adults are at high risk for strokes because of the risk of a thromboembolic event from the plaque. Renal dysfunction can occur as a result of chronic hypertension; the intimal lining of the renal arteries is damaged over time, which leads to renal artery stenosis and decreased renal perfusion. Cardiac disease is a common result of chronic hypertension. Isolated systolic hypertension is a common consequence of aging but not a result of end-organ damage. Genetic factors determine familial hypercholesterolemia and cannot be caused by end-organ damage.

When differentiating the characteristics of depression, delirium, and dementia, the nurse recognized which of the following as an indicator of delirium? a. Sudden onset b. Recent loss c. Insidious d. Life change

a. Sudden onset Delirium can occur suddenly. Recent loss or life changes can precipitate depression. Dementia can be insidious, slow, and occur over the course of several years.

At 10 PM, an older adult resident attempts to climb over the bedrails. Which intervention should the nurse implement first? a. Talk to the resident about his behavior. b. Call the primary health care provider and ask for a sedative. c. Apply a vest restraint on the resident. d. Get a companion to keep him in the bed.

a. Talk to the resident about his behavior. The resident is expressing a need that the nurse can potentially determine with gentle questioning. Pharmacological intervention can be necessary but should not replace careful evaluation and management of the underlying cause. Simply restraining the client will not address the underlying problem, and the imposition of restraints can trigger delirium. Applying a restraint is the last resort, and the nurse must consider the problems that accompany the application of restraints before doing so. Placing a companion in the room can be an effective method of keeping the resident safe if the companion can determine and meet the resident's needs.

An older client in an adult day care program tells the nurse that, "I'm very stressed because another neighbor passed away." The most therapeutic response by the nurse is: a. "What do you mean by 'stressed'?" b. "Tell me what you did when your other neighbor passed away." c. "Are you worrying about your own death?" d. "Let's get involved in some activities and not think about sad things."

b. "Tell me what you did when your other neighbor passed away." Application of what one has learned from previous situations can help dissipate the intensity of stress. Denial of the stressful event and focusing upon blessings or happiness will not lessen the stress and may in turn intensify it. While it is appropriate to ask the client to clarify what they are saying, it does not help in this situation. This is not necessarily the time to initiate a conversation about the client's feelings about death since doing so is likely to increase the level of stress.

Which older adult is most likely to have healthy mental well-being? a. The older adult who grieves over the loss of a spouse for 2 years but is traveling again b. The older adult who exhibits long periods of depression with occasional manic episodes c. The older adult who has lost two friends in a war, has had three failed marriages, and is bankrupt d. The older adult who has been treated for chronic depression and whose brother killed himself 1 year ago

a. The older adult who grieves over the loss of a spouse for 2 years but is traveling again The older adult who grieves after suffering a major loss for 2 years, which is a length of time for grief that is within normal limits, is beginning to enjoy life again. This individual is most likely to have normal mental health because he or she has worked through the grief and has had the strength to resume normal activities. The older adult who exhibits long periods of depression with occasional manic episodes has clinical indicators of bipolar disorder. The older adult, who has lost two friends in a war, has had three failed marriages, and is bankrupt, is unlikely to enjoy normal mental health; this older adult's life displays an inability to cope effectively with tragedy, relationships, and personal matters. The older adult who has been treated for chronic depression and whose brother killed himself 1 year ago is at risk for suicide and is unlikely to have normal mental health.

Which population(s) is(are) most at risk for developing HIV? (Select all that apply.) a. Those over the age of 50 years b. Sexually active older women c. Those who are cognitively impaired d. Any older sexually active person e. Sexually active bisexual men

a. Those over the age of 50 years b. Sexually active older women d. Any older sexually active person Older adults who are sexually active are at risk for HIV, AIDS, and other sexually transmitted diseases. People older than 50 years of age are approximately one-sixth as likely to use condoms during sex. Older women who are sexually active are at high risk for HIV, AIDS, and other sexually transmitted infections from an infected partner, resulting, in part, from normal age changes of the vaginal tissue—a thinner, drier, friable vaginal lining that makes viral entry more efficient. Gay and bisexual men remain disproportionally affected by HIV. Being cognitively impaired does not put one at high risk.

The home health care nurse observes that the older male client is confined by his daughter-in-law to his room. When the nurse suggests that he walk to the living room and join the family, he says, "I'm in everyone's way; my daughter-in-law needs me to stay here." The most important action for the nurse to take is to a. suggest appropriate resources to the client and daughter-in-law, such as respite care and a senior citizens' center. b. suggest to the client and daughter-in-law that they consider a nursing home for the client. c. say nothing, because it is best for the nurse to remain neutral and wait to be asked for help. d. say to the daughter-in-law, "Confining your father-in-law to his room is inhuman."

a. suggest appropriate resources to the client and daughter-in-law, such as respite care and a senior citizens' center. Assisting clients and families to become aware of available community support systems is a role and responsibility of the nurse. Suggesting committing the client to a nursing home is a premature action on the nurse's part. Observing that the client has begun to be confined to his room makes it necessary for the nurse to intervene legally and ethically. Telling the daughter-in-law that the confinement is inhuman is both incorrect and judgmental.

How should the nurse respond when an older adult asks, "How much alcohol is good for you?" a. "Alcohol isn't good for you so avoid it as a general rule." b. "Experts in the field recommend only one regular sized drink a day." c. "It's been said that red wine has health benefits but that doesn't mean drink a whole bottle." d. "If you are only drinking on special occasions, limit yourself to two drinks."

b. "Experts in the field recommend only one regular sized drink a day." Clinically significant adverse effects can occur in some individuals consuming as little as two to three drinks per day over an extended period. Because of the increased risk of adverse effects from alcohol use, the National Institute on Alcohol Abuse and Alcoholism (NIAAA) defines "at-risk drinking" for men and women aged 65 years and older as more than one drink per day. The other options do not address the client's question.

Which of the following questions will best assess the ability of the lesbian, gay, bisexual, transgender (LGBT) older couple to successfully adjust to the challenges of aging? a. "How long have you been in this relationship?" b. "Have you experienced prejudice and discrimination in your life?" c. "As a couple are you financially secure?" d. "Do you as a couple share similar religious beliefs?"

b. "Have you experienced prejudice and discrimination in your life?" Some research has suggested that this population may adapt more successfully to old age as a result of successful coping over a lifetime with discrimination and prejudice. While the other options may be factors, the coping skills of the individuals are of primary importance in adjusting to challenges.

An older adult client diagnosed with dementia resides with an adult daughter. When the home care nurse visits, the daughter tearfully tells the nurse that her parent scratched her hand and cursed at her when she was attempting to provide care. She states, "I don't know why my partner hates me and wants to hurt me." How will the nurse respond to the client's daughter? a. "Let's think about what you may have done to anger your father." b. "Let's try to figure out what your father was trying to say with his behavior." c. "Scratching is usually a sign of untreated pain. Do you think your father is in pain?" d. "Maybe you should consider having a home health provide your father's physical care."

b. "Let's try to figure out what your father was trying to say with his behavior." Dementia often interferes with the person's communication and the ability to understand and express thoughts and feelings. The focus needs to be on what the person is attempting to communicate through behavior. Behavioral manifestations are not necessarily signs of anger in persons with dementia. Although behavioral manifestations frequently are seen in persons with untreated pain, this is not always true. The issue here is not necessarily the individual who is providing the care but perhaps the care activity itself. It is appropriate for the daughter to provide care for her father.

The nurse is providing an educational session to new staff employees, and the topic is abuse of the older client. Which client is most typically a victim of abuse? a. A 69-year-old man who has newly diagnosed cataracts b. A 90-year-old woman who has advanced Parkinson's disease c. A 70-year-old woman who has early diagnosed Lyme disease d. A 74-year-old man who has moderate hypertension

b. A 90-year-old woman who has advanced Parkinson's disease Elder mistreatment is a complex phenomenon that includes elder abuse and neglect. Elder abuse includes physical, sexual, or psychological abuse, misuse of property, and violation of rights. The typical abuse victim is a woman of advanced age with few social contacts and at least one physical or mental impairment that limits her ability to perform activities of daily living. In addition, the client usually lives alone or with the abuser and depends on the abuser for care.

The nurse identifies which risk factor(s) for arthritis? (Select all that apply.) a. Men b. American Indian c. Advanced age d. Steroid use e. History of Paget's disease

b. American Indian c. Advanced age Advanced age and American Indian ethnicity are risk factors for the development of arthritis. Women tend to be more at risk while long-term steroid use and a history if Paget's disease are risk factors for osteopenia.

When working with a bereaved individual, what is the goal of nursing interventions? a. Assist the individual to go through the stages of grief work in the optimal order. b. Assist the individual to attain a healthy adjustment to the loss experience. c. Encourage the individual to talk about his or her feelings about the deceased individual. d. Offer support and advice about how to successfully achieve grief work.

b. Assist the individual to attain a healthy adjustment to the loss experience. The goal of nursing interventions when working with bereaved individuals is to help them adjust in a healthy manner. There is no optimal order in which to experience grief. Not all individuals are able to talk about their feelings, nor is it helpful for everyone. The role of nursing is to offer support, but not advice.

Which of the following is a true statement about heart disease in older adults? a. Myocardial infarction (MI) has many of the same symptoms in older clients as in middle-aged persons. b. Both excessive urination at night and decreased urination can be signs of heart failure (HF). c. Any exertion on the part of an older adult client with heart disease can bring on another heart attack. d. A person with HF is likely to have trouble breathing, except when lying down.

b. Both excessive urination at night and decreased urination can be signs of heart failure (HF). Because the heart is an ineffective pump in HF, both excessive urination at night and decreased urination can occur in older adults. Nocturia occurs in HF when the heart is unable to maintain adequate renal blood flow in the performance of daily activities; then, during the night when the client's lower extremities are elevated for sleep, the heart is able to perfuse the kidneys with the assistance of increased venous return owing to the elevated extremities. Inadequate urine production is due to inadequate perfusion from an ineffective pump. The classic presentation of angina pectoris in older clients is often absent in what is known as a silent MI, with only mild discomfort, perhaps even limited to nausea or "heartburn" as the only symptom. Failure to engage in cardiac rehabilitation exercises is more likely to result in another MI or be an aggravation of HF than ordinary exertion. A person with HF is more likely to have difficulty breathing except when the trunk is upright (orthopnea).

Which of the following indicate a person is effectively coping? (Select all that apply.) a. Avoids avoidance b. Confronts realities c. Focuses on solutions d. Redefines problems e. Acknowledge the loss

b. Confronts realities c. Focuses on solutions d. Redefines problems People who cope well confront reality and deal with situations; acknowledging their loss. They focus on the solution and redefine the problem. People who cope well avoid avoidance.

Which of the following should the nurse use to assess a nonverbal older adult for delirium? a. Cranial nerves XI and XII b. Confusion Assessment Method c. MMSE-2 d. Controlled Word Association Test

b. Confusion Assessment Method The Confusion Assessment Method (CAM) is a tool for measuring delirium in clients who are intubated or nonverbal. Assessing the accessory (CN XI) and hypoglossal (CN XII) cranial nerves provides clues about the client's ability to swallow. The nurse uses the Controlled Word Association Test to assess for a neurologic cause of an older adult's cognitive dysfunction. This tool is an index of frontal lobe functioning and provides an assessment of executive function, including the client's frontal lobe functioning and his or her ability to refrain from distraction and perseveration. The MMSE-2 is a valid and reliable tool to assess cognitive function; however, it is unable to pinpoint discrete areas of neurologic dysfunction.

An older client who has end-stage pulmonary disease decides to accept care from the palliative care nurse. This older adult will most likely benefit from the palliative care nurse in which client needs of Weisman's needs for the dying? a. Closure b. Control c. Composure d. Cohesiveness

b. Control The dying client is most likely to benefit from the care of the palliative care nurse by affording the client as much control as possible, providing effective nursing care for symptom control and by providing continuity of care as the palliative care team directs total client care. In providing control, the nurse asks the client to determine activities and how time is spent. Palliative care can indirectly benefit the client by providing a better quality of life at the end of life, but palliative care does not provide for closure. Nurses use countercoping techniques to help the client maintain composure. Cohesiveness is not one of the needs of a client who is dying.

The children of an older woman ask the nurse for advice about helping their mother heal after her husband's (their father's) death. Which strategy should the nurse share with the family? a. Appoint one family member to take her on outings. b. Coordinate family expressions of care and concern. c. Have each child plan a long trip with her assistance. d. Take her to community events to meet other people.

b. Coordinate family expressions of care and concern. The nurse suggests that the family work together to provide extended expressions of caring and concern for their mother; many small expressions of concern and caring from several sources help the bereaved gain the strength and confidence needed to survive a huge loss. Multiple small gestures are more likely to help build strength and confidence than a few large gestures. One person is unlikely to provide enough support for the bereaved, and this strategy can potentially imply that only one person is concerned. Helping a widow meet new people can be unsuitable; she may be uninterested or unwilling to attend events for meeting new people. In addition, she may feel that the family is trying to find a replacement for the deceased to ease the family's burden. However, the family can offer to accompany her to such events.

Which information will the nurse manager include when discussing the major differentiation between delirium and dementia with novice nurses? (Select all that apply.) a. The delirious client learns to make up answers to hide their confusion. b. Delirium requires increased monitoring at night. c. The client diagnosed with dementia generally looks frightened. d. Dementia results in a steady decline in cognitive abilities. e. Delirium is characterized by fluctuations in alertness.

b. Delirium requires increased monitoring at night. d. Dementia results in a steady decline in cognitive abilities. e. Delirium is characterized by fluctuations in alertness. The correct options accurately describe the conditions of delirium and dementia. The other statements are false; it is the client experiencing dementia who will over the course of the illness learn to confabulate to cover up their memory losses, and the delirious client is more likely to show fear through facial expressions.

Which of the following is(are) true statement(s) about depression or depression therapy? (Select all that apply.) a. An older adult who lived through the depression is unlikely to develop depression. b. Depression in older adults is often neglected or assumed to be a condition of aging. c. Serotonin-reuptake inhibitors are rarely used to resolve depression in 2 weeks. d. Depression in the older population is considered a significant public health issue. e. Alcohol abuse is a common comorbidity.

b. Depression in older adults is often neglected or assumed to be a condition of aging. d. Depression in the older population is considered a significant public health issue. e. Alcohol abuse is a common comorbidity. Depression remains underdiagnosed and undertreated in the older population and is considered a significant public health issue. Depression in older adults can be effectively treated. Unfortunately, it is often neglected or assumed to be a condition of aging that one must "learn to live with." Depression is often comorbid with alcohol abuse. Older adults who have endured the horrors of the mid-20th century (e.g., the Great Depression, the Holocaust, and World War II) are as prone to depression as other older adults, but they can consider it shameful to acknowledge depressive feelings. Serotonin-reuptake inhibitors, usually the drug of choice for depression, can be unsuitable for a specific individual. All antidepressant medications must be closely monitored for side effects and therapeutic response. Only about one-third of depressed older adults achieve remission with any single agent.

An older adult comes to the emergency department after falling at home, and reports "I can't walk without losing my balance." Which steps should the nurse implement for this client? a. Arrange for a transfer immediately to the radiology department. b. Determine symptom onset or when the fall occurred. c. Organize the reperfusion recombinant tissue plasminogen activator (tPA) infusion. d. Perform a comprehensive neurologic assessment.

b. Determine symptom onset or when the fall occurred. The nurse determines when the symptoms first appeared or the time of the fall to determine whether sufficient time is left to administer reperfusion tPA; if indicated, tPA must be administered within 3 hours of symptom onset. A client with clinical indicators of a stroke will need a computed tomographic (CT) scan to differentiate between a thrombotic stroke and a hemorrhagic stroke; the type of stroke will determine the therapeutic course. The time of symptom onset is a vital piece of information that must be determined before the client is referred to the radiology department because tPA is usually administered in the radiology suite. Administering tPA can be contraindicated for this client; therefore, the preparation of this infusion is delayed until the type of stroke and the plan of care are determined. The nurse will not have enough time to complete a comprehensive assessment and thus will perform a focused assessment in preparation for the trip to radiology.

A 78-year-old client who is dying of colon cancer with metastases to the liver is refusing to eat or drink. He is alert and oriented and states that he has no desire to eat, which is causing the family great distress. In order to best address the client and family, what should the nurse do? a. Explains the family's concern to the client. b. Educates the family that this is normal behavior in this situation. c. Contacts the physician for an order for enteral feeding. d. Contacts the dietitian for feeding supplements.

b. Educates the family that this is normal behavior in this situation. The nurse should educate the family that this is a normal part of the dying process, and the nurse should not pressure the client, contact the physician for enteral feeding, or contact the dietitian for feeding supplements. Because the client is expressing a desire not to eat, his wishes should be honored. Essential to the facilitation of self-esteem is the premise that the values of the client must figure significantly in the decisions that will affect the course of dying. Whenever possible, the nurse can have the person decide when to groom, eat, wake, sleep, and so on.

A client loses her husband as a result of a sudden myocardial infarction, and she blames herself for not recognizing the warning signs. She has also begun to withdraw from family and friends. Which client outcome associated with her loss should the nurse use to plan care? a. Meets her daily responsibilities b. Expresses feelings of guilt, fear, anger, or sadness c. Assesses the causes of the dysfunctional grieving processes d. Identifies problems connected to anticipatory grief

b. Expresses feelings of guilt, fear, anger, or sadness The nurse plans care that will help this client resolve her grief and will work to accomplish this by determining a suitable client outcome—the ability of the client to express feelings of guilt, fear, anger, or sadness. Being able to express herself in this manner is part of the work of grief. Expecting the client to meet her daily responsibilities is a nursing intervention suitable for dysfunctional grieving. Assessing the causes of the dysfunctional grieving processes is a nursing intervention suitable for the grieving client. Identifying problems connected to anticipatory grief is a client outcome suitable for anticipatory grief.

Which of the following behavior modifications should the nurse instruct a client to accomplish to help reduce the risk factors for an occurrence of a stroke. (Select all that apply.) a. Increase the intake of green, leafy vegetables. b. Stop smoking. c. Control blood pressure. d. Increase physical activity. e. Avoid obesity.

b. Stop smoking. c. Control blood pressure. d. Increase physical activity. Stopping smoking, avoiding obesity, keeping blood pressure under control, and incorporating physical activities are all modifiable risk factors. Increasing the intake of green leafy vegetables does not, in itself, decrease the risk of stroke; however, they are part of a healthy diet if the client is not taking an anticoagulant medication.

Which of the following statements are true about caregivers or caregiving? (Select all that apply.) a. Approximately 75% of all caregivers are male with an average age of 49 years. b. Family caregivers are children, spouses, and other family and friends. c. Caregiving can have serious negative effects on mental and physical health. d. The average duration of a caregiver's role is 10 years. e. Hispanic (non-White, non-African-American) caregivers have the lowest reported prevalence of caregiving. f. Caregiving can present financial burdens. g. Women who are family caregivers are 2.5 times more likely than noncaregivers to live in poverty.

b. Family caregivers are children, spouses, and other family and friends. c. Caregiving can have serious negative effects on mental and physical health. f. Caregiving can present financial burdens. g. Women who are family caregivers are 2.5 times more likely than noncaregivers to live in poverty. The most common caregiver arrangement is that of an adult female child providing care to an older female parent, with approximately 75% of all caregivers being female. Family caregivers include children, spouses, and other family and friends. Caregiving can have serious negative effects with approximately 40% to 70% of caregivers having clinically significant symptoms of depression. The average duration of a caregiver's role is 4 years. Hispanic (non-White, non-African-American) caregivers have the highest reported prevalence of caregiving at 21%. Caregiver prevalence rates among other racial/ethnic groups are: Asian-Americans, 20.3%; African-American, 19.7%; White, 16.9%. Financial burdens are also found with caregiving.

The health care provider believes an older woman has approximately 6 weeks to live. After 2 months, the family remains at the bedside but, in the last few days, are becoming increasingly impatient and irritable. This pattern is least indicative of which of the following statements? a. Family is experiencing anticipatory grief for the older adult. b. Family desires that the client be relieved of her misery. c. Anticipatory grieving can fail to attenuate acute grief upon death. d. Grievers deal more easily with known losses at known times.

b. Family desires that the client be relieved of her misery. The family is not demonstrating impatience and irritability because they want her death to take place and want her to be relieved of her misery; they are displaying these behaviors because of the emotional ups and downs resulting from the waiting. The remaining three statements are true.

The nurse should suggest which of the following to a spouse of a client with dementia who has displayed inappropriate sexual behavior to decrease the occurrence? (Select all that apply.) a. Intimate relations b. Hug c. Kiss d. Touch e. Cuddle a stuffed animal

b. Hug c. Kiss d. Touch Inappropriate sexual behavior may be triggered by unmet intimacy needs or may be symptoms of an underlying physical problem, such as a urinary tract or vaginal infection. Encouraging family and friends to touch, hug, kiss, and hold hands when visiting may help meet the client's touch and intimacy needs and decrease inappropriate sexual behavior. Holding a stuffed animal may be a healthy substitute for missing human contact.

A nurse measures an older adult's blood pressure on the right arm and notes a reading of 150/100. The nurse waits 5 minutes and measures the blood pressure again in the right arm and obtains a reading of 152/100. What is the next action by the nurse? a. Immediately contact the medical provider. b. Measure the blood pressure in the left arm. c. Measure the blood pressure in sitting and standing positions. d. Document the findings in the medical record; elevated blood pressures are normal in older adults.

b. Measure the blood pressure in the left arm. When an abnormal blood pressure reading is obtained, it is necessary to do two measurements, 5 minutes apart, confirmed in the contralateral arm. Doing orthostatic measurements is not indicated in this situation. A blood pressure reading above 140/90 is considered abnormal for the general population. However, a systolic reading up to 150 mm Hg is now acceptable for those at least 65 years of age, except for those with diabetes.

Which one of the following older adults is most likely to need preretirement counseling to avoid significant concerns in retirement? a. African American woman who is a certified public accountant b. Mexican American woman who receives cash for cleaning services c. Middle-aged man who has a history of type 1 diabetes mellitus d. Older male clerk who works for the Department of Homeland Security

b. Mexican American woman who receives cash for cleaning services Older adults with the most need for retirement planning are usually the same people who are least likely to receive it. The Mexican American woman will most likely need preretirement counseling to avoid significant problems in retirement. She has three characteristics indicating a need for retirement planning: (1) her sex is female, (2) her job implies a lack of education or training, and (3) she does not pay into Social Security or federal taxes. Thus, she can potentially fail to pay into the federal system for a sufficient length of time to be eligible for Social Security and Medicare benefits in retirement. In addition, her job is unlikely to offer a 401k plan. Although the African American woman is a member of a minority group, she is a professional and likely to earn a significant income in a finance-related business. Her occupation and education places her well to receive significant retirement planning. The middle-aged man has one factor potentially associated with the need for retirement planning—poor health. The older clerk has one factor potentially associated with the need for retirement planning—he is likely to be a low-level employee.

A nursing student is preparing a presentation on arthritis. The nursing student knows that differences between osteoarthritis (OA) and rheumatoid arthritis (RA) include: (Select all that apply.) a. Both OA and RA have an acute onset in older adults. b. OA is a localized process where RA may be systemic. c. OA usually impacts distal interphalangeal joints, RA impacts proximal interphalangeal joints. d. Both OA and RA present with joint stiffness lasting 20 to 30 minutes after rest. e. Initial treatment of both OA and RA is usually nonpharmacological using heat or exercise.

b. OA is a localized process where RA may be systemic. c. OA usually impacts distal interphalangeal joints, RA impacts proximal interphalangeal joints. OA has an insidious presentation; RA has an acute presentation. OA presents with joint stiffness which resolves in less than 20 minutes, RA presents with joint stiffness that lasts more than 20 to 30 minutes. OA is initially treated with nonpharmacological treatments such as heat or exercise, RA is treated with medications (MARDs) immediately after diagnosis.

Which assessment finding of an older adult living in an assisted-living facility indicates the highest risk for suicide? a. Liver failure is due to alcohol abuse; older adult is popular at meals. b. Older adult declines company; is preoccupied with lethal weapons. c. Refuses to allow a large, extended family to help him. d. Older adult had an overdose of acetaminophen 20 years ago; is in a sewing group.

b. Older adult declines company; is preoccupied with lethal weapons. The older adult who prefers to be alone and is preoccupied with lethal weapons has two risk factors for suicide. This individual warrants close observation for additional risk factors and verbalization and indicators of future suicide attempts. The nurse should also increase the frequency of observations and account for his whereabouts at all times. The individual who has a serious illness and a history of alcohol abuse has two risk factors for suicide. However, this older adult also relishes social interaction, which is an indication that suicide is less likely to be imminent or even in the individual's thoughts. The older adult who will not accept help from the family exhibits a potential risk factor for suicide or is an exceedingly proud individual who wants to be self-sufficient. History of a suicide attempt is a risk factor for suicide; however, the acetaminophen overdose could have been accidental.

An older adult says to the nurse, "I don't know why I can't handle booze like I used to when I was younger." The nurse's response is based on what knowledge? a. Older adults develop higher blood alcohol levels due to age-related changes in the neurological system. b. Older adults develop higher blood alcohol levels due to age-related changes that alter absorption and distribution of alcohol. c. Older adults develop higher blood alcohol levels due to slowed reaction times. d. Older adults develop higher blood alcohol levels due to cognitive changes.

b. Older adults develop higher blood alcohol levels due to age-related changes that alter absorption and distribution of alcohol. Age-related changes such as increased body fat, decreased lean muscle mass, and decreased total body water content alter absorption and distribution of alcohol, increasing blood alcohol levels. Age-related neurological changes do not impact blood alcohol levels. Slowed reaction time does not impact blood alcohol levels. Cognitive changes do not impact blood alcohol levels; furthermore, not all older adults experience cognitive changes.

An African American 58-year-old man in good health has a blood pressure at 120/73 mm Hg at his annual physical examination. Which of the following is the best goal for the nurse to use to assist him in maintaining his health and wellness into older age? a. Alter modifiable risk factors. b. Prevent cardiovascular disease. c. Recognize disease in early stage. d. Maintain tight glycemic control.

b. Prevent cardiovascular disease. The nurse assists this adult in maintaining health and wellness by helping him prevent cardiovascular disease; as an older African American man, he has a high risk of stroke, hypertension, and diabetes mellitus. Furthermore, he is more likely to die of a stroke or heart attack than other people in the United States. To help him prevent cardiovascular disease, the nurse assists him with lifestyle modifications. Specific recommendations the nurse can make include getting regular exercise; learning the warning signs of heart attack and stroke; maintaining a normal weight; controlling blood pressure; eating a well-balanced, low-fat, no-added-salt diet; and avoiding smoking. Altering modifiable risk factors is a subgoal to preventing cardiovascular disease. Learning the early warning signs of disease is a subgoal to preventing cardiovascular disease. Although he has no clinical indicators of hyperglycemia, he is at risk for developing diabetes mellitus, which is an important subgoal of preventing cardiovascular disease for an African American man.

An older adult is recovering from a bowel resection in the intensive care unit but remains intubated and on a mechanical ventilator. Which of the following should the nurse implement to help prevent delirium from developing in this client? a. Assess the client's cognition using the Single Question in Delirium (SQID) tool. b. Provide uninterrupted periods of rest and sleep. c. Maintain adequate sedation and pain management. d. Cover the client's eyes with protective ophthalmic ointment.

b. Provide uninterrupted periods of rest and sleep. Providing uninterrupted periods of rest and sleep is a challenge for the nurse in intensive care. Because of the nature of the clients' illnesses, nurses administer medications and treatments and perform invasive procedures on a 24-hour basis, leaving clients little time for rest. Many clients become delirious in the intensive care unit because the noise, activity, brightness, and disturbance tend to persist around the clock, which contribute to delirium. Clients lose their sources for maintaining orientation and stability; that is, bright lighting at all times, as well as unfamiliar and abrupt increases in noise, can lead to a disruption in the circadian rhythm. In addition, clients in intensive care are more likely to receive multiple medications, and medications that are potentially harmful can aggravate the client's cognitive difficulties. Because this client is intubated and on mechanical ventilation, the nurse cannot apply the Single Question in Delirium (SQID); the patient is unable to perform adequately. Besides, assessing for dementia is not a prophylactic measure. Sedation and pain management, although often needed in the intensive care unit, can contribute to delirium. Covering the eyes of a client in intensive care with ointment can be necessary to prevent corneal damage; however, it is likely to contribute to delirium because the client will be unable to see clearly.

Which of the following reactions to the loss of a spouse or long-term partner is a unique example of older adult male bereavement? a. Withdrawing from friends and family b. Remarrying within months of the loss c. Focusing on "doing" rather than "feeling" d. Experiencing moderate to severe depression

b. Remarrying within months of the loss Bereaved males may be more socially and emotionally vulnerable. Widowers adapt more slowly than widows to the loss of a spouse and often remarry quickly. The remaining options reflect reactions that are typically seem in both grieving men and women.

A new nurse in a long-term care facility is caring for a client diagnosed with Parkinson's disease (PD). The nurse should note that which one of the following actions is likely to be observed during the assessment? a. Changing facial expression b. Resting hand tremors c. Frequent movement d. Fast movements

b. Resting hand tremors Clients diagnosed with PD demonstrate major motor signs that include resting tremor, muscular rigidity, bradykinesia, and akinesia. Clients with PD exhibit tremors at rest in their hands, arms, legs, feet, and jaw.

Which of the following is a true statement about joints in older adults? a. Osteoarthritis (OA) is an inflammatory joint disorder. b. Surgical joint replacement can effectively manage OA pain. c. Joint damage in OA is reversed with medication. d. Very old clients should avoid joint replacement surgery.

b. Surgical joint replacement can effectively manage OA pain. Surgical joint replacement can effectively manage OA-related pain. OA is a degenerative joint disease, whereas rheumatoid arthritis (RA) is an inflammatory process. Medications are used to control the pain of OA. The joint damage cannot be reversed except through joint replacement surgery. Surgical joint replacements are recommended even for those who are very old.

Which of the following statements is true about the role of grandparents? a. The usefulness of grandparents declined with the advent of the industrial age. b. Today, many grandparents are the primary caregivers of their grandchildren. c. The value of grandparents is to provide gifts to younger family members. d. Traditionally, parents are subordinate to the grandparents in caregiving.

b. Today, many grandparents are the primary caregivers of their grandchildren. Grandparents have always had an important role to play and have become more important than ever in recent decades. An increasing number of parents have been unable to provide necessary care for their children as a result of personal problems; often, grandparents fill the gaps. Grandparents provide continuity, family tradition, and accumulated wisdom. Parents are still expected to be the primary caregivers.

Although the older adult man who was forced to retire from law enforcement has multiple physical complaints, the primary care health care provider finds nothing abnormal. After the man tells the nurse that his girlfriend just left him, which of the following is the priority nursing intervention to complete before the older adult leaves? a. Ask him how he plans to cope with his loss. b. Use direct questions about access to firearms. c. Collaborate with his provider for antidepressants. d. Allow him to express himself by intently listening.

b. Use direct questions about access to firearms. The nurse's priority intervention is to ask him directly about his access to firearms; he has familiarity with guns, and the risk factors for suicide in older adults include male gender, physical complaints of unknown causes, and having suffered a recent loss. Asking him how he plans to cope with his loss is a reasonable intervention for the nurse to include in the plan of care for this older adult in light of his risk factors for suicide. Collaborating with his provider for antidepressants is a reasonable intervention for the nurse to include after a comprehensive assessment of this older adult. Allowing him to express himself by intently listening is a reasonable intervention for the nurse to include because it helps the nurse establish a trusting, caring relationship with this older adult.

After completing an admission assessment on a client who recently experienced a stroke, the nurse should choose which of the following nursing diagnoses as a priority? a. Risk for injury b. Altered thought process c. Altered cerebral perfusion d. Decreased mobility

c. Altered cerebral perfusion Altered cerebral perfusion is the priority diagnosis. Altered cerebral perfusion may be caused by an interruption in blood flow such as occlusive disorder, hemorrhage, cerebral vasospasm, or cerebral edema. It is important for the nurse to monitor cognitive status and vitals for clients experiencing altered cerebral perfusion. The client is at risk for injury due to the effects of the stroke; however it is not the priority diagnosis. This client may suffer from altered thought processes due to cerebral damage from the stroke; however, this is not the priority diagnosis. This client may experience a decrease in mobility such as hemiparesis; however, it is not the priority diagnosis.

Which of the following is a true statement about psychotic behavior in older adults? a. Usually, hallucinations in older clients are the result of psychological conflicts. b. Illusion, delusion, and hallucination are different terms for the same phenomenon. c. An older adult with psychotic behavior should be assessed for a variety of causes. d. Regardless of the cause, dissimilar hallucinations are treated with similar therapies.

c. An older adult with psychotic behavior should be assessed for a variety of causes. The nurse assesses an older adult who is exhibiting psychotic behavior by searching for a reason from a wide variety of potential causes for the behavior. For example, neuroleptic medications can cause extrapyramidal side effects, which can result in movement disorders that are similar to psychotic behavior. Hallucinations in older clients are usually the result of physical disorders, dementias, or sensory function loss. A delusion is a belief that is maintained, although facts can prove that it is incorrect. A hallucination or illusion is the sensory perception of a stimulus that does not exist in the external world. Treatments for hallucinatory states vary according to the cause.

The nurse notices that an older female nursing home resident is not eating and that her heart rate is faster than usual. Which should the nurse do to determine if pneumonia is a potential cause of the change in her status? a. Obtain a specimen for aerobic blood cultures. b. Promptly send the resident for a chest x-ray examination. c. Analyze sputum for color, texture, and volume. d. Compare tympanic temperature to the baseline.

c. Analyze sputum for color, texture, and volume. Sputum cultures are indicated to assess a resident for pneumonia. Sputum is a sensitive and specific clinical indicator of pneumonia for older adults in nursing homes. If pneumonia is causing this resident's anorexia and tachycardia, then her sputum should be cloudy, colored, and thick, especially if the resident is dehydrated, which indicates an infection. Blood cultures are likely to show no growth unless the resident has severe sepsis. A chest x-ray study is a nonsensitive, nonspecific diagnostic tool for determining the presence of pneumonia in an older adult. Fever can be a late indicator of infection for an older adult.

After the older adult dies, the brother who has a history of alcohol abuse upsets the family by going on a drinking binge instead of attending the funeral. Which of the following is the best description of the brother's behavior? a. Personality disorder b. Disrespectful attitude c. Disenfranchised grief d. Chronic grief

c. Disenfranchised grief When a family is in discord, a grieving member can be unable to or consider him or herself permitted to express grief by socially acceptable means. The brother's behavior is most likely a grief reaction, although it could be indicative of a personality disorder. The brother can feel that the most respectful thing he can do for the family members is to stay out of their way. The brother has suffered an acute loss. The described behavior is not noted in any of the other options.

An older man is being abused by his daughter, a single working mother of four children, with whom he lives. The nurse investigates and learns that the abuse is due to situational stress. Which of the following interventions should the nurse implement to address the short-term crisis? a. Immediately remove him from his daughter's home. b. Encourage the daughter to work with social services. c. Arrange respite care or day care for the client. d. Place the client in a long-term care facility.

c. Arrange respite care or day care for the client. By relieving the daughter of some responsibilities, respite care is likely to be beneficial for the older adult and his daughter; it can help reduce tension. Unlike children, abused older adults cannot be removed from their situations without their permission. Helping the daughter manage the situational stress would be more effective. Encouraging the daughter to work with social services can help teach her more effective and harmless ways of solving problems, but it does not address the short-term crisis. Placing the client in a long-term care facility may eventually be necessary but improving the living situation within the client's family would be better.

After an acute exacerbation of chronic obstructive pulmonary disease (COPD), the nurse prepares an older adult for discharge to home. Which is the most important client teaching for the nurse to include for the prevention of hospitalizations for exacerbations of COPD? a. Ease breathing by sitting upright. b. Use low-flow oxygen for dyspnea. c. Avoid sick people and wash hands. d. Eat nutrient- and calorie-dense foods.

c. Avoid sick people and wash hands. The nurse helps the client with COPD maintain health and wellness by preventing infection. To accomplish this, the nurse instructs the client to avoid people with contagious illnesses to reduce exposure to communicable diseases and to wash hands frequently to reduce exposure to microorganisms as potential pathogens. Following these instructions will help the client avoid hospitalizations for COPD; a pulmonary infection can have a devastating impact on a client who has compromised pulmonary reserves. Fluid and exudates accumulate in the lungs to decrease oxygenation and ventilation, and the client with COPD is less able to cough and expel sputum. The nurse teaches the client to sit upright to ease breathing for transient dyspnea that occurs after exertion or while eating. This technique, however, is unlikely to prevent a hospitalization for the client with an exacerbation of COPD. The client with COPD regularly uses oxygen for dyspnea as prescribed. Oxygen provides symptomatic relief of dyspnea but does not prevent hospitalizations for exacerbated COPD. Eating nutrient- and calorie-dense food is also important. Clients with COPD work very hard at breathing; therefore, the clients need the calories and nutrition to supply fuel for the work of breathing. In addition, clients with COPD should eat these foods because eating them in sufficient quantities to meet their needs is often difficult; therefore, the food they do eat must contain many calories and nutrients. Nutritional issues are not the most important aspect of preventative therapy for clients with COPD; an infection is more likely to cause a more devastating problem.

An older man in a cardiac rehabilitation exercise class refuses to participate in the cool-down phase of the activity; consequently, 2 minutes later, he passes out but quickly regains consciousness. Which instruction does the nurse include in client teaching to reinforce the importance of cooling down after exercising to this man? a. Cardiac output diminishes with age. b. Mobility capacity decreases with age. c. Baroreceptor function diminishes with age. d. Sensory perception diminishes with age.

c. Baroreceptor function diminishes with age. During exercise, the body shunts blood to the skeletal muscles to supply enough oxygen to meet the increased metabolic demands of the muscles. If the exercise is suddenly withdrawn, however, the blood temporarily pools in the skeletal muscles, and the older adult loses consciousness from transient hypotension. Baroreceptor responsiveness declines with age; therefore, the body does not respond as readily to the need for changes in blood pressure. The cool-down period compensates for this effect. An acute problem such as losing consciousness as a result of decreased cardiac output should appear during the "real" exercise. The man is mobile enough to participate in the exercise program. Gradual sensory alteration does not account for the acute episode.

Which of the following statements is true about rheumatoid arthritis (RA)? a. Strikes unilaterally. b. Affects more men than women. c. Can affect body systems other than the joints. d. Morning stiffness in RA lasts less than 30 minutes.

c. Can affect body systems other than the joints. RA is a systemic disease and can affect body systems other than joints; this statement is true. Women are affected more often than men. RA strikes the same parts of the body on both sides and affects joints in a symmetrical pattern. RA can affect body systems other than joints and can cause general fatigue and malaise and attack systems other than joints. Morning stiffness in RA it lasts longer than 30 minutes.

Which of the following nursing interventions are suitable as a last resort treatment for gout? a. Nonsteroidal anti-inflammatory drugs (NSAIDs) b. Liquid paraffin hand baths c. Colchicine by mouth d. Hyaluronic acid injections

c. Colchicine by mouth Colchicine is indicated in the treatment of an acute gout attack when other interventions have failed to manage the pain. NSAIDs can be used in pain management in all forms of arthritis. Liquid paraffin hand baths can be used to relieve pain in OA and RA. Hyaluronic acid injections are used by some to relieve the pain of OA in the knee. Salicylates should not be used in gout because they can exacerbate an attack.

Which classic sign of an acute myocardial infarction (AMI) can be absent in an older man with an AMI? a. Vague complaints b. Epigastric burning c. Crushing chest pain d. Dyspnea and fatigue

c. Crushing chest pain Gripping chest pain, radiating to the shoulder, is typically seen in younger adults, but not always in older adults. Instead, an older adult may be experiencing a silent MI. Older adult with an AMI expresses vague complaints such as fatigue, weakness, and dyspnea. Older adults with an AMI are seen with atypical complaints such as epigastric burning or pain.

The nurse sees an older adult diagnosed with osteoarthritis (OA) who presents with a low-grade fever. The client tells the nurse that the pain is changing; it is worse at night and in the shoulder muscles. Which of the following does the nurse perform to prevent complications of this client's condition? a. Assess all joints for swelling and redness. b. Obtain blood specimens for blood cultures. c. Direct the client to report temporal or scalp pain. d. Direct the client to apply moist heat for 20 minutes.

c. Direct the client to report temporal or scalp pain. The older adult exhibits clinical indicators of polymyalgia rheumatica (PMR), and a serious complication of PMR is giant cell arteritis (GCA). The nurse instructs the client to report scalp and temporal pain because they are early indicators of GCA. As a complication of PMR, the client exhibits clinical indicators of PMR that include severe pain and stiffness of muscles, including the back, buttocks, and thighs. PMR is not a disease that affects the joints. Blood cultures are not indicated for PMR because it is not an infection. Because PMR is an autoimmune, inflammatory disorder, applying heat is more likely to aggravate the client's condition. Effective treatment for PMR includes low-dose steroids. However, low-dose steroids are unrelated to preventing complications of PMR.

The son of an older adult couple ends his life suddenly and violently. The husband proceeds with living as usual. After 1 year, the wife remains in seclusion and is hospitalized for dehydration. Which steps should the nurse implement to help improve the wife's mental health and wellness? a. Encourage additional fluids and social activity. b. Instruct the husband to display empathy for her. c. Establish an empathic relationship with her. d. Ask social services for a survivor's support group.

c. Establish an empathic relationship with her. This couple is at high risk for chronic grief because of the nature of their son's death. The nurse can help this older woman work toward better mental health by establishing a trusting and empathic relationship with her to encourage expressions about her son. Afterward, the nurse can pose probing questions to determine the best plan of care to help this older adult resolve or reframe enough grief to allow improved functioning. Encouraging fluids can be a reasonable nursing intervention for this woman; however, encouraging social activity without a complete assessment and without being in the environment of a trusting relationship is unlikely to help improve functioning. The husband can be displaying disenfranchised grief and be unable to help until his grief is managed. A survivor's support group can be helpful to survivors of a loved one's suicide; however, expecting this older adult to attend such a group is unrealistic until the nurse establishes a trusting relationship with her.

The nurse in a rehabilitation center is caring for a client diagnosed with new-onset stroke with right-side hemiparesis. Which intervention should the nurse implement when caring for this client? a. Orders a two-person assist with a transfer. b. May need to incorporate repetition. c. Gives the client a dry erase board. d. Raises all four side rails.

c. Gives the client a dry erase board. Right-side hemiparesis involves a left-side brain injury. The left side of the brain controls speech and language. By giving the client a dry erase board, he or she can communicate easier initially after the stroke. People who have this type of hemiparesis experience difficulty talking. With only one side affected, the nurse should be able to transfer the client alone. Clients with left-side hemiparesis have with short term memory, often repetition must be incorporated into client care. The raising of all four side rails up would be considered a restraint.

A nurse is teaching an older adult who is experiencing an acute attack of gout. Which of the following instructions should the nurse include in the teaching? (Select all that apply.) a. Walk barefooted as much as possible. b. Take ASA to relieve pain. c. Increase fluid intake to 2 L/day. d. Avoid foods high in purine. e. Avoid alcoholic beverages.

c. Increase fluid intake to 2 L/day. d. Avoid foods high in purine. e. Avoid alcoholic beverages. Strategies to optimize positive outcomes include the avoidance of risk-elevating drugs or of foods that are high in purine and alcohol, both of which increase uric acid levels and to increase fluid intake to flush uric acid through the kidneys. Individuals who are having an acute attack of gout should not take salicylates for pain; ASA is a salicylate. Walking barefooted would not be therapeutic nor a safe practice.

An older adult who has Alzheimer disease exhibits new behaviors including shouting in the hallways and hallucinations. Rank the following nursing interventions in order, beginning with the first intervention the nurse should implement in response to the new behavior. a. Review the medication list for potential causes. b. Plan nursing care to promote a trusting relationship. c. Look for the likely causes for psychotic manifestations. d. Consult with health care provider about medications.

c. Look for the likely causes for psychotic manifestations. a. Review the medication list for potential causes. b. Plan nursing care to promote a trusting relationship. d. Consult with health care provider about medications. The nurse's first task is to identify the likely causes of psychotic behaviors to provide a framework for planning suitable nursing interventions. Second, as a potential cause of the new behaviors, the nurse reviews the medication list and looks for new medications, missed or increased doses, polypharmacy, and medications likely to cause psychotic behavior. Third, after identifying possible pharmacological reasons, the nurse consults with the health care provider to consider adjustments to the pharmacotherapy. Finally, to supplement the removal of offending medications, the nurse promotes a trusting relationship with the older adult by expressing respect and concern.

An older adult is diagnosed to be in the early stage of Alzheimer's disease. The diagnosis is made on the presence of which of the following outcomes? (Select all that apply.) a. A decline from a previous level of functioning b. Impaired judgement c. Mild memory loss d. A gradual decline in cognitive abilities e. Easily frustrated

c. Mild memory loss d. A gradual decline in cognitive abilities Diagnosis of AD requires that there has been a decline in the person's cognitive functioning from a previous level, that it developed slowly, that the changes are "greater than expected for the person's age and educational background," and that these changes can be documented with standardized neuropsychological testing. Mild memory loss is noted in the early stage of this disease. The other options are indicative of the middle stage of the disease.

The nurse recognizes which of the following displays may indicate hyperactive delirium? a. Lethargy b. Withdrawn behavior c. Nonpurposeful repetitive movements d. Decreased psychoactive activity

c. Nonpurposeful repetitive movements Clients with hyperactive delirium often wander and have nonpurposeful repetitive movements. Lethargy and withdrawn behavior are both indicative of hypoactive delirium. Clients with hyperactive delirium have increased psychoactive activity, not decreased.

Which nursing intervention best addresses the need for social support demonstrated by an older adult couple who will be assuming responsibility for the raising of two grandchildren? a. Facilitating a support group for children being raised by grandparents b. Helping the grandparents express their feeling regarding this unexpected role change c. Offering a monthly parenting class for this cohort of grandparents d. Suggesting couple's therapy to assist in managing any new stress on their marriage

c. Offering a monthly parenting class for this cohort of grandparents Primary care grandparents with a network of social support seem to experience fewer negative consequences, but instrumental supports such as assistance with child rearing are often lacking. Education and training programs and support groups are valuable resources and nurses can be instrumental in developing and conducting these types of interventions. While the remaining options are appropriate, they do not focus on the needs that are identified as currently unaddressed.

A nurse plans for the discharge of a 75-year-old client diagnosed with of osteoporosis. Which of these actions would the nurse consider first to best address safety needs? a. Avoid stressful situations. b. Schedule an annual DXA/DEXA scan. c. Remove clutter from the floors of the home. d. Encourage consumption of a high-protein diet.

c. Remove clutter from the floors of the home. Individuals with osteoporosis are very high risk for falls. The most serious health complication of osteoporosis is the morbidity and mortality associated with a falls. There is no evidence that stress impacts osteoporosis. The recommendation for a DXA/DEXA scan is every 2 years. There is no evidence that a high protein diet is important for an individual with osteoporosis.

The nurse prepares an older adult diagnosed with osteoarthritis (OA) for discharge. Which instruction does the nurse include in client teaching to maintain safety for this client? a. Take ibuprofen rather than opioid analgesics. b. Increase rest periods to slow disease progression. c. Report joint instability to the health care provider. d. Avoid stretching the affected joint during exercise.

c. Report joint instability to the health care provider. As OA progresses, the joint deteriorates and can become unstable, thereby increasing the risk of falls. The joint stability will not improve without physical therapy or surgery; therefore, the client needs to report instability to the health care provider. Although ibuprofen is much less likely to cause dizziness, hypotension, or sedation, nonsteroidal anti-inflammatory agents such as ibuprofen are poor analgesic choices for older adults; they can aggravate hypertension and impair renal blood flow. The nurse avoids recommending increased rest because rest contributes to stiffness. Stretching is an important form of exercise for older adults with OA; it helps maintain joint flexibility and range of motion.

Which of the following statements is not true about respite care? a. Respite care allows the caregiver to take a break from caregiving for various periods of time. b. Respite care may be provided in institutions, in the home, or in other community settings. c. Respite care services are utilized frequently in the course of caregiving. d. Respite care services are sometime paid for by nonprofit agencies.

c. Respite care services are utilized frequently in the course of caregiving. Linking caregivers to community resources, such as respite care, adult day programs, and financial support resources, is important. These community services, when available, can alleviate much of the stress of caregiving but are utilized infrequently or very late in the course of caregiving. Some government and nonprofit agencies offer free respite help but most respite services are paid privately by the family.

When the older woman who is close to death asks the family to leave after short visits and acts withdrawn in their presence, the family becomes distraught. Which of the following does the nurse include in family teaching to explain the client's behavior? a. She is preoccupied with her own death. b. She must have unresolved family issues. c. She can be experiencing anticipatory grief. d. Her body prepares for death in this manner.

c. She can be experiencing anticipatory grief. Anticipatory grief occurs before the death and can be experienced by the client or the potential survivors. When the client who is dying experiences anticipatory grief, the individual detaches from the environment into a state sometimes described as psychological death. The person is no longer involved in day-to-day activities and enacts a premature death. Preoccupation with death is acute grief, but since the death has not occurred, preoccupation cannot describe this client. The client might have unresolved issues with the family, but this behavior is an unusual way to express it. The body prepares for death through the deterioration of organ system functioning; for example, the kidneys stop producing urine, the client stops drinking and eating, and the extremities become cold, among other functions.

As the wife of a university president, an older woman met exciting people and traveled extensively until her husband died. Besides losing an intimate partner, the nurse identifies that this woman is most likely to grieve for the loss of which area of her life? a. Self-confidence b. Economic security c. Status in community d. Intellectual stimulation

c. Status in community After the loss of her husband, this older adult will most likely mourn the loss of her status in the community; no longer is she a wife or the wife of a community leader. After her husband's death, the opportunities for university-related travel and social occasions will most likely disappear from her life. After extensive travel and sophisticated social stimulation, this older adult is likely to be self-confident and is likely to have acquired life skills from her experiences. The president of a university is likely to have planned for retirement and is likely to have left his survivors with an adequate estate. This woman will most likely to be able to provide intellectual stimulation in her own life, although the stimulation from the university-related activities will likely decrease.

Which of the following is a true statement about loss, dying, and death for older adults? a. Men and women tend to respond similarly to the loss of a spouse. b. Visions on the part of a person who has lost a spouse are not normal grief reactions and should be regarded as signs of underlying defects. c. The grieving process is not rigidly structured. d. Bereaved persons regain their normal capability approximately 6 months after loss, and regressive behavior after that time should be discouraged.

c. The grieving process is not rigidly structured. Men and women do not always respond similarly to the loss of a spouse. The grieving process is not rigidly structured and is not always predictable. Visions, hallucinations, and an inability to communicate in a logical, sustained manner on the part of a person who has lost a spouse are not normal grief reactions and should be regarded as signs of underlying defects; all these reactions are common in the first several months of bereavement. A bereaved person ordinarily begins to recover personal control and capabilities after approximately 6 months; at first, recovery is sporadic and interspersed with periods of depression.

Which of the following is a true statement about osteoporosis? a. Osteoporosis is indicative of an underlying health problem. b. The most common site for related fractures is in long bones. c. Men of all races have the highest risk for osteoporosis. d. A high risk of death follows an osteoporosis-related fracture.

d. A high risk of death follows an osteoporosis-related fracture. The most serious outcomes of osteoporosis are fractures and are associated with high mortality. Osteoporosis can be a natural-occurring consequence of aging. The vertebrae, pelvis, and wrist are the most frequent sites for OA fractures. In developed countries only 2% to 8% of men and 9% to 38% of the women are diagnosed with osteoporosis.

Which of the following is a true statement concerning suicide among older adults? a. Older adults and younger adults manifest a suicidal intent in a similar manner. b. Older African American women have the highest risk of suicide among older adults. c. Ethics require that the nurse respects a person's intent to terminate his or her own life. d. A major crisis experienced by the client can contribute to the risk of suicide.

d. A major crisis experienced by the client can contribute to the risk of suicide. Major crises or transitions, such as retirement or relocation to an assisted living or nursing facility, can contribute to the risk of suicide. Putting personal affairs in order, distributing possessions, making a will, or saying something similar to, "I won't be around much longer," can indicate a risk for suicide in a young person but can be a rational and mature act in older age. Men in all countries have a higher suicide rate, and White men are more likely to evaluate their worth solely in terms of their present economic productivity. Chinese American women aged 65 years and older have the highest suicide rate of all women older than the age of 65 years in the United States (National Alliance on Mental Illness, 2011). Health care professionals are obligated to prevent the destruction of life as a permanent solution to what may be a temporary problem.

The community health nurse delivers a program to middle-aged adults about retirement planning and wants to them to choose the year of their retirement. Which is the most important area on which the participants should focus to ease the transition to retirement? a. Kind of legacy they want to leave behind b. Type of setting for their personal residence c. Location of convenient health care services d. Ability to maintain a stable standard of living

d. Ability to maintain a stable standard of living The most significant factors contributing to a smooth transition into retirement are health, income, and social involvement; therefore, the nurse helps the participants focus on financial issues to begin retirement planning, thereby establishing the future retiree's ability to maintain health, income, and social involvement. By choosing a year for retirement, individuals can estimate their retirement income and consult specialists in retirement planning, such as the employee's human resources department, to determine retirement benefits. Defining the kind of legacy is a secondary factor and less likely to contribute to a smooth transition into retirement. The location of retirement and the location of health care services are also less likely to contribute to a smooth transition into retirement.

A home health nurse is completing an admission on a client who recently experienced a transient ischemic attack (TIA). During the assessment, the client begins to report a severe headache and numbness in the left arm. Which action should the nurse take initially? a. Give the client a dose of acetaminophen. b. Determine if the client has a history of migraine headaches. c. Assure the client the symptoms will resolve within 24 hours. d. Call 9-1-1.

d. Call 9-1-1. To best assure the client's safety, the home health nurse should immediately call 9-1-1. The client has a history of TIA which is a risk factor for a stroke. While it is true that a TIA is ischemic but clinically different from a stroke in that all of the neurologically associated symptoms begin to resolve within 24 hours, the client is at risk for a stroke. Acetaminophen would not be advised. Migraine headaches don't generally present with numbness of an extremity.

An older woman seeks advice from the nurse about preventing further bone loss after being diagnosed with osteopenia. To achieve the woman's goal, which of the following client teachings should the nurse provide to enhance the activity of the osteoblasts? a. Limit sodium intake. b. Refrain from alcohol use. c. Eat high-fiber foods. d. Engage in weight-bearing exercise.

d. Engage in weight-bearing exercise. In osteopenia, bone metabolism is unbalanced because the action of osteoclasts is greater than the action of osteoblasts. To treat osteopenia effectively, the balance between the activities of the bone cells must be shifted to more osteoblast (bone-building) activity; increasing osteoblast activity helps reduce bone loss and, at the same time, helps gain bone density. Weight-bearing exercise stimulates osteoblasts to build bone through the application of opposing forces on the bone and helps achieve the woman's goal by increasing physical activity (to stem bone loss) and by generating more bone (to gain bone density). Sodium impairs calcium absorption; therefore, the nurse instructs her to limit sodium intake to reduce bone loss. Alcohol impairs calcium absorption; therefore, the nurse instructs her to avoid alcoholic beverages. Fiber inhibits calcium absorption.

An older adult client who had radical surgery for oral cancer is refusing to see visitors and is losing weight, despite aggressive nutrition therapy. The nurse assesses the client for ineffective coping, related to dysfunctional grieving. Which of the following client outcomes of nursing care is the most important to implement in response to his mental health status? a. Is able to discuss how usual coping mechanisms are overwhelmed. b. Performs daily self-feedings through a gastrostomy tube. c. Effectively uses nonverbal forms of communication. d. Exhibits self-confidence in regaining a sense of control.

d. Exhibits self-confidence in regaining a sense of control. The most important element of the nursing plan of care for this older adult is to create and strengthen self-confidence to improve his sense of control; doing so is likely to help him effectively manage the other aspects of his health care. The nurse helps create and improve this self-confidence by observing for strengths and integrating them into his daily care and by responding with empathy and encouragement to his expressions of fears, emotions, and desirable goals. Helping this client gain self-confidence is the most important outcome because this man has clinical indicators for depression—social isolation and weight loss. Before this client can benefit from discussing his stressors or from client teaching, the nurse must establish a trusting, caring relationship and build some self-confidence because, at this point, this individual feels hopeless and believes that he has no control. The client displays a lack of readiness for expressions about emotions, coping, or his stressors; by enhancing his self-confidence, the nurse prepares him to discuss coping mechanisms and stressors. This client also displays a lack of readiness for learning a new psychomotor activity. Performing daily self-feeding is an outcome that gains importance as the day for discharge approaches. Effectively using nonverbal forms of communication is important for basic communication; however, he displays a lack of readiness for receiving help to achieve this outcome.

Which of the following statements is true about the mental health of older adults? a. Nurses should discourage denial and regression so older adults can directly face underlying causes of anxiety. b. Anxiety is easily distinguished from depression, dementia, and the effects of disease or medication. c. Compulsive rituals surrounding toileting and sleep are signs of a serious mental disorder. d. The nurse avoids antianxiety medications without an assessment for factors associated with anxiety.

d. The nurse avoids antianxiety medications without an assessment for factors associated with anxiety. Without an adequate assessment, medications can exacerbate a problem. Denial and regression may be necessary to enable an older person to cope with underlying stressors. Depression, dementia, disease, and medications can produce anxious behavior, and the resultant anxiety can be manifested in a similar manner, regardless of the cause. Compulsive rituals can be a way of coping with challenges leading to anxiety.

Which of the following approaches to hygienic care is beneficial for a client diagnosed with dementia? a. Schedule the client's full shower at 7 AM, three mornings every week. b. Have a team give the bath with each member washing a different body area. c. Wash the perineal region first to remove potentially infectious material. d. Explain each step as you go and keep the client covered as much as possible while bathing.

d. Explain each step as you go and keep the client covered as much as possible while bathing. A person with dementia can interpret undressing for bathing as an assault. It should be performed in a way that minimizes the intrusive and exposing aspects and maintains trust between the person and only one caregiver. From the point of view of the well-being of the client, bathing is rarely an emergency that it must be performed at a time when the client is not receptive. Stimulation should be kept simple and focused, and alarming the client should be avoided. The most sensitive and intimate areas should be washed last, after trust has been established between the client and the nurse, which may have to be done anew at every encounter. From an infection-control standpoint, washing occurs from clean to dirty areas.

Which of the following statements is not true of older adult relationships? a. After 50 years of marriage, a couple can face new and severe challenges to their relationship. b. Older adults often hold their families together by arranging get-togethers and documenting the family's history and rituals. c. Losing a brother or sister brings an older adult face-to-face with his or her own death. d. For older adults, friends can never take the place of family.

d. For older adults, friends can never take the place of family. Friendships can provide the commitment and support that is sometimes lacking in family relationships. Physical and psychosocial changes related to aging, such as declining health, reduced income, and mismatched needs, may cause a severe strain even for a couple who has been together for 50 years or more. A person who holds his or her family together by arranging get-togethers and documenting the family's history and rituals is known as a kin-keeper. The impact of the loss of a brother or sister, not only at the time of the death but also when a younger survivor reaches the age at which the lost sibling died, can be quite disruptive.

The home health care nurse suspects that a client's spouse is experiencing caregiver burden. How would the nurse assess for this condition? a. Referring the family to a social services agency b. Waiting for the caregiver to talk about the stress c. Obtaining feedback from the client about the caregiver d. Gathering assessment data from the caregiver and the client

d. Gathering assessment data from the caregiver and the client Although caregiving is a means to "give back" to a loved one and can be a source of joy, it is also stressful. Caregiver burden can occur when a client is significantly dependent on the caregiver for personal and health care needs. The nurse gathers data from the client and the caregiver to determine the caregiver's stressors and coping abilities and withholds making any referrals until the assessment is complete and the plan of care is in place. Because the nurse suspects caregiver burden, the nurse fulfills the duty to the client and family by approaching the family with the concern, gathering assessment data, and planning care. The nurse does not expect the client to assess the coping abilities of the caregiver, because assessment is part of the nursing process and should not be delegated.

What makes nursing support of caregivers so important for health care in the United States? a. Family members providing care in the home are the best caregivers. b. Eighty percent of caregiving takes place in the home of the older adult. c. The health care system reimburses families for caregiving from Medicare. d. Informal caregiving saves the health care system enormous sums of money.

d. Informal caregiving saves the health care system enormous sums of money. The value of caregiving is estimated at $470 billion; therefore, informal caregiving is a huge savings for the health care system that it might potentially be unable to provide in the event that it was called to do so. Although family members can be the most convenient and are the least expensive, they do not necessarily make the best caregivers. Family members provide 80% of the caregiving for older adults. Caregiving is not an expense for which Medicare reimburses the family.

Which of the following statements is true about Parkinson disease (PD)? a. Drinking large amounts of alcohol can relieve symptoms of essential tremor. b. Motor tremors and slow movement accompany severe cognitive impairment. c. Lewy body dementia (LBD) is the most common form of dementia. d. It is advised to take levodopa-carbidopa on an empty stomach.

d. It is advised to take levodopa-carbidopa on an empty stomach. Clients taking levodopa-carbidopa (Sinemet) must take it on an empty stomach (i.e., 30 to 60 minutes before a meal or 45 to 60 minutes after a meal) in order for it to be effective. It is given on a set schedule to prevent fluctuation in symptoms. Drinking small amounts of alcohol can relieve symptoms of essential tremor, although heavy drinking should be avoided. The majority of persons with PD remain alert and intelligent, but motor difficulties in facial expression and speech can give a false impression of cognitive impairment. LBD, which can occur in some clients with PD symptoms, is the second most common form of dementia. It accounts for 15% to 20% of all dementias.

Which of the following statements is true about cardiopulmonary disease in older adults? a. Chronic obstructive pulmonary disease COPD can be reversed with proper treatment. b. Chest radiographic studies are a reliable indicator of whether pneumonia is present in an older client. c. Persons older than 65 years should receive Pneumovax annually. d. Mouth hygiene is essential to prevent and treat pneumonia.

d. Mouth hygiene is essential to prevent and treat pneumonia. Bacteria from the mouth can migrate into the lower respiratory tract and cause infection. COPD cannot be reversed. For a debilitated person at the beginning of the course of infection or in dehydration, the chest x-ray study is often falsely negative. While the Pneumovax vaccine is most often a one-time dose, the older adult's health care provider may recommend a second dose at a later time.

Which is the most important element for older adults to have for enhancing the transition into retirement? a. Good health b. Private pension c. 401k retirement plan d. Preretirement planning

d. Preretirement planning Good health is always a desirable state; however, if an older adult in poor health plans for retirement, then the transition to the changes of retirement can be smoother if the planning accounts for health challenges. With preretirement planning, private pensions are not obligatory. With preretirement planning, 401k retirement plans are not obligatory; however, these plans are effective saving plans for retirement; they allow employees to save pretax dollars. Preretirement planning is the most important aspect of a smooth transition into retirement because it affords the individual a chance to prepare for losses associated with retirement such as income, interpersonal communication, health insurance, status, influence, and other issues. Health, social, and financial planning help the older adult adapt to expected or sudden retirement.

After living with osteoporosis for 2 years, an older adult's bone density scan shows no improvement, despite consistent bisphosphonate therapy. Which intervention should the nurse implement to reduce bone loss for this older adult? a. Add tai chi or yoga exercises. b. Instruct the client to drink fortified milk. c. Increase weight-bearing exercises. d. Review the client's daily nutritional habits.

d. Review the client's daily nutritional habits. Reviewing the older adult's nutritional habits can reveal clues about potential dietary contributors to bone loss from excessive sodium, alcohol, caffeine, or carbonated beverage intake. In addition, the nurse also confirms that the client avoids smoking and a sedentary lifestyle that contribute to bone loss. Tai chi or yoga, drinking fortified milk, and increasing weight-bearing exercises can all help increase bone density.

The actions of the family members of an older adult who just died are chaotic, and they are unable to decide on a funeral home. Which recommendation should the nurse implement? a. Help them make a list of the problems. b. Provide a list of preferred funeral homes. c. Allow them privacy to work it out alone. d. Suggest they call someone who can help.

d. Suggest they call someone who can help. A characteristic of a good coper is one who has good communication with others; however, immediately after the older adult's death, this family is ineffectively coping with the loss. To facilitate the family with decision-making, the nurse asks one family member to consider calling another person who will likely help the family face the reality of the death. After making the suggestion, the nurse ensures that the family has enough time for holding behaviors, to prepare the body if they wish, and to express their grief in privacy and in their own way. Effective coping includes a focus on the solutions rather than on the problems. The nurse avoids recommending funeral homes, which could be a potential conflict of interest. The family can be unable to solve the problem alone because of ineffective coping in the immediate mourning period.

Which of the following interventions is recommended for an older adult in the final stages of dying? a. Apply an electric blanket to keep the client warm. b. Lower the head of the bed and turn the head to the side. c. Decrease the number of visitors. d. Support the preservation of energy.

d. Support the preservation of energy. Conserving energy should be a focus in the care of a client in the final stage of dying. Completing only the necessary activities of daily living (ADLs) would be an example. An electric blanket should not be used; it can increase the client's distress by overheating. Elevating the head of the bed and turning the client's head to the side is a recommended intervention to help clear uncomfortable respiratory congestion. Nurses should not withhold visitors; the client needs to have closure, as well as the family.

Which of the following characteristics of rheumatoid arthritis (RA) are unlike those of osteoarthritis (OA)? a. Myalgia and stiffness b. Joint pain that is curable c. Crepitus and instability d. Systemic and symmetrical

d. Systemic and symmetricala OA is not a systemic disease, nor does it affect joints symmetrically. Myalgia and stiffness are characteristics of PMR; however, myalgia is uncharacteristic of RA and OA. Joint pain is characteristic of both RA and OA, but only OA is curable through joint replacement. RA is a systemic disease and affects joints symmetrically; therefore, these are clinical indicators of OA, not RA.

A widowed grandmother is about to assume the role of custodial parent for her 6-year-old grandchild. Which intervention has priority when preparing the grandmother for long-term success in this new role? a. Reviewing the developmental milestones of childhood b. Identifying local sources of child counseling services c. Discussing the common challenges of parenting a 6-year-old d. Teaching stress management and relaxation techniques

d. Teaching stress management and relaxation techniques Grandmothers raising grandchildren reported the most stress, intrafamily strain, and perceived problems in family functioning with the stress increasing over time. The introduction of the skills identified in the correct option will serve to improve and maintain the family's quality of life. While the remaining options are appropriate, they lack attention to the long-term, fundamental barrier to a successful transition into this new role.

Which of the following statements is true about cognitive impairments in older adults? a. Loss or interruption of sleep can lead to delirium. b. Confusion is a normal and unavoidable consequence of aging. c. Older clients who are agitated often have a lower cognitive status than those who are quietly sitting. d. The Confusion Assessment Method (CAM) should be administered on admission to detect delirium.

d. The Confusion Assessment Method (CAM) should be administered on admission to detect delirium. The Confusion Assessment Method (CAM) or a similar instrument should be administered to a client at admission to ascertain the client's baseline cognitive status. The loss or interruption of sleep, in of itself, does not often lead to delirium. It can potentiate delirium in the presence of other factors. Confusion or delirium is not a normal consequence of aging but an indicator of a potentially underlying problem. The hypoactive subtype of delirium can be associated with a worse prognosis than with the hyperactive subtype; it is easily overlooked.

An older adult arrives at the emergency department with a probable diagnosis of a hemorrhagic stroke. The nurse understands, based on the client's age, that the most likely cause is which one of the following? a. Intracranial hemorrhage b. Decreased cardiac output c. Thrombosis d. Uncontrolled hypertension

d. Uncontrolled hypertension Hemorrhagic strokes are primarily caused by uncontrolled hypertension and less often by malformations of the blood vessels (e.g., aneurysms). Although the exact mechanism is not fully understood, it appears that chronic hypertension causes a thickening of the vessel wall, microaneurysms, and necrosis. When enough damage to the vessel accumulates, it is at risk for rupture. The spontaneous rupture may be large and acute or small with a slow leak of blood into the adjacent brain tissue. In many cases, blood ruptures or seeps into the ventricular system of the brain with damage to the affected tissue through necrosis or death of brain tissue. Hemorrhagic strokes are more life threatening but occur less frequently than ischemic strokes. Decreased cardiac output does not cause this type of hemorrhage. A thrombosis is not related to this type of hemorrhage.

An older female resident lowers her voice and tells the nurse that another female resident is looking at her behind her back and is going to make her move tonight with a male staff member. Which ideas should the nurse include in the response to this individual? a. The staff receives training in ethics. b. Validate the woman's impression. c. Avoid suspicious, paranoid thinking. d. Use the call bell if she becomes frightened.

d. Use the call bell if she becomes frightened. Telling the resident to use the call bell if she becomes frightened offers assurances to the client that she will be protected, but it neither confirms her suspicions nor makes a promise that cannot be kept. Replying that the staff receives training in ethics sounds as if the nurse is arguing in defense of the male staff member and does not help alleviate the resident's fear, which can lead her to suspect that the nurse, is also a part of the plot. Validating the woman's impression contributes to the resident's suspicions; in addition, the nurse increases professional liability risks by speaking about another resident in a negative manner. Telling the resident to avoid suspicious, paranoid thinking only aggravates the struggle for control.


Related study sets

Ch 15: The system of Internal Membranes (Secretion)

View Set

Day 2 Vocabulary: browser, bookmarks, & folder structure

View Set

Kidneys and Urinary System Review (powerpoints)

View Set

HDFS 2200 Complete Study Guide--Chapters 1 through 12

View Set

Psyc 523 - Chapter 3: Skills Approach

View Set

Chapter 26: Nursing Management: Upper Respiratory Problems

View Set

NBSN 8004 - Biostatistics: Module 4 (RR/OR; Sensitivity, Specificity; Survival Analysis)

View Set

Pharmacology - Prep U - Chapter 38

View Set